Why do all primes $n>3$ satisfy $,309mid 20^n-13^n-7^n$ Unicorn Meta Zoo #1: Why another...

Is Bran literally the world's memory?

What is /etc/mtab in Linux?

What is the numbering system used for the DSN dishes?

What to do with someone that cheated their way though university and a PhD program?

Test if all elements of a Foldable are the same

What helicopter has the most rotor blades?

What is the term for extremely loose Latin word order?

What is ls Largest Number Formed by only moving two sticks in 508?

How to begin with a paragraph in latex

What's parked in Mil Moscow helicopter plant?

Putting Ant-Man on house arrest

What is the ongoing value of the Kanban board to the developers as opposed to management

Why I cannot instantiate a class whose constructor is private in a friend class?

Are these square matrices always diagonalisable?

What is the purpose of the side handle on a hand ("eggbeater") drill?

What was Apollo 13's "Little Jolt" after MECO?

All ASCII characters with a given bit count

How did Elite on the NES work?

Can gravitational waves pass through a black hole?

My admission is revoked after accepting the admission offer

Philosophers who were composers?

Is there a verb for listening stealthily?

Does using the Inspiration rules for character defects encourage My Guy Syndrome?

Why did Europeans not widely domesticate foxes?



Why do all primes $n>3$ satisfy $,309mid 20^n-13^n-7^n$



Unicorn Meta Zoo #1: Why another podcast?
Announcing the arrival of Valued Associate #679: Cesar ManaraPrime factor of $A=14^7+14^2+1$Number of divisors excluding set of primes SCalculating $e^{3x} pmod{27}$Help understanding number theory definitionWhat's wrong with this proof of the infinity of primes?Unclear on why Meissel's approach to counting primes worksIf $a^n+n^bmid c^n+n^d$ for every $n$ then $c=a^k$ and $d=kb$ .What does x equivalent to 2 mod 15 mean?Olympiad problem: Prove that there are infinitely many primes which divide the sequence ${ S(k) } _{kin mathbb{N}}$Primes of Atlantis: a definition and a problemWhat is an irreducible prime?












6












$begingroup$


Solve the following... $309|(20^n-13^n-7^n)$ in $mathbb{Z}^+$.
I invested lotof time to it and finally went to WolframAlpha for help by typing...
Solve $309k=20^n-13^n-7^n$ over the integers. It returned the following...enter image description hereenter image description here Note that all the primes are generated here. Can someone please explain why? Thanks! EDIT: the formulas suggested in the answerare too complicated, but this is so simple!










share|cite|improve this question









New contributor




Shamim Akhtar is a new contributor to this site. Take care in asking for clarification, commenting, and answering.
Check out our Code of Conduct.







$endgroup$












  • $begingroup$
    Is it false that it generates all the primes?
    $endgroup$
    – Shamim Akhtar
    11 hours ago










  • $begingroup$
    It is easy to picture why there might be a solution for all prime $n$. However, the interesting part of the pattern is that $n=5cdot 5$ and $n=5cdot 7$ both appear, yet neither $n=3cdot 3$ nor $n=5cdot 5$ appear. Which odd composites admit solutions?
    $endgroup$
    – Mark Fischler
    11 hours ago










  • $begingroup$
    There are no even numbers ..
    $endgroup$
    – Shamim Akhtar
    10 hours ago








  • 3




    $begingroup$
    It seems like $n equiv pm1 pmod 6$.
    $endgroup$
    – md2perpe
    10 hours ago










  • $begingroup$
    Yeah right but why do only specific composites arrive?
    $endgroup$
    – Shamim Akhtar
    10 hours ago
















6












$begingroup$


Solve the following... $309|(20^n-13^n-7^n)$ in $mathbb{Z}^+$.
I invested lotof time to it and finally went to WolframAlpha for help by typing...
Solve $309k=20^n-13^n-7^n$ over the integers. It returned the following...enter image description hereenter image description here Note that all the primes are generated here. Can someone please explain why? Thanks! EDIT: the formulas suggested in the answerare too complicated, but this is so simple!










share|cite|improve this question









New contributor




Shamim Akhtar is a new contributor to this site. Take care in asking for clarification, commenting, and answering.
Check out our Code of Conduct.







$endgroup$












  • $begingroup$
    Is it false that it generates all the primes?
    $endgroup$
    – Shamim Akhtar
    11 hours ago










  • $begingroup$
    It is easy to picture why there might be a solution for all prime $n$. However, the interesting part of the pattern is that $n=5cdot 5$ and $n=5cdot 7$ both appear, yet neither $n=3cdot 3$ nor $n=5cdot 5$ appear. Which odd composites admit solutions?
    $endgroup$
    – Mark Fischler
    11 hours ago










  • $begingroup$
    There are no even numbers ..
    $endgroup$
    – Shamim Akhtar
    10 hours ago








  • 3




    $begingroup$
    It seems like $n equiv pm1 pmod 6$.
    $endgroup$
    – md2perpe
    10 hours ago










  • $begingroup$
    Yeah right but why do only specific composites arrive?
    $endgroup$
    – Shamim Akhtar
    10 hours ago














6












6








6


4



$begingroup$


Solve the following... $309|(20^n-13^n-7^n)$ in $mathbb{Z}^+$.
I invested lotof time to it and finally went to WolframAlpha for help by typing...
Solve $309k=20^n-13^n-7^n$ over the integers. It returned the following...enter image description hereenter image description here Note that all the primes are generated here. Can someone please explain why? Thanks! EDIT: the formulas suggested in the answerare too complicated, but this is so simple!










share|cite|improve this question









New contributor




Shamim Akhtar is a new contributor to this site. Take care in asking for clarification, commenting, and answering.
Check out our Code of Conduct.







$endgroup$




Solve the following... $309|(20^n-13^n-7^n)$ in $mathbb{Z}^+$.
I invested lotof time to it and finally went to WolframAlpha for help by typing...
Solve $309k=20^n-13^n-7^n$ over the integers. It returned the following...enter image description hereenter image description here Note that all the primes are generated here. Can someone please explain why? Thanks! EDIT: the formulas suggested in the answerare too complicated, but this is so simple!







elementary-number-theory






share|cite|improve this question









New contributor




Shamim Akhtar is a new contributor to this site. Take care in asking for clarification, commenting, and answering.
Check out our Code of Conduct.











share|cite|improve this question









New contributor




Shamim Akhtar is a new contributor to this site. Take care in asking for clarification, commenting, and answering.
Check out our Code of Conduct.









share|cite|improve this question




share|cite|improve this question








edited 8 hours ago









Bill Dubuque

214k29198660




214k29198660






New contributor




Shamim Akhtar is a new contributor to this site. Take care in asking for clarification, commenting, and answering.
Check out our Code of Conduct.









asked 11 hours ago









Shamim AkhtarShamim Akhtar

469




469




New contributor




Shamim Akhtar is a new contributor to this site. Take care in asking for clarification, commenting, and answering.
Check out our Code of Conduct.





New contributor





Shamim Akhtar is a new contributor to this site. Take care in asking for clarification, commenting, and answering.
Check out our Code of Conduct.






Shamim Akhtar is a new contributor to this site. Take care in asking for clarification, commenting, and answering.
Check out our Code of Conduct.












  • $begingroup$
    Is it false that it generates all the primes?
    $endgroup$
    – Shamim Akhtar
    11 hours ago










  • $begingroup$
    It is easy to picture why there might be a solution for all prime $n$. However, the interesting part of the pattern is that $n=5cdot 5$ and $n=5cdot 7$ both appear, yet neither $n=3cdot 3$ nor $n=5cdot 5$ appear. Which odd composites admit solutions?
    $endgroup$
    – Mark Fischler
    11 hours ago










  • $begingroup$
    There are no even numbers ..
    $endgroup$
    – Shamim Akhtar
    10 hours ago








  • 3




    $begingroup$
    It seems like $n equiv pm1 pmod 6$.
    $endgroup$
    – md2perpe
    10 hours ago










  • $begingroup$
    Yeah right but why do only specific composites arrive?
    $endgroup$
    – Shamim Akhtar
    10 hours ago


















  • $begingroup$
    Is it false that it generates all the primes?
    $endgroup$
    – Shamim Akhtar
    11 hours ago










  • $begingroup$
    It is easy to picture why there might be a solution for all prime $n$. However, the interesting part of the pattern is that $n=5cdot 5$ and $n=5cdot 7$ both appear, yet neither $n=3cdot 3$ nor $n=5cdot 5$ appear. Which odd composites admit solutions?
    $endgroup$
    – Mark Fischler
    11 hours ago










  • $begingroup$
    There are no even numbers ..
    $endgroup$
    – Shamim Akhtar
    10 hours ago








  • 3




    $begingroup$
    It seems like $n equiv pm1 pmod 6$.
    $endgroup$
    – md2perpe
    10 hours ago










  • $begingroup$
    Yeah right but why do only specific composites arrive?
    $endgroup$
    – Shamim Akhtar
    10 hours ago
















$begingroup$
Is it false that it generates all the primes?
$endgroup$
– Shamim Akhtar
11 hours ago




$begingroup$
Is it false that it generates all the primes?
$endgroup$
– Shamim Akhtar
11 hours ago












$begingroup$
It is easy to picture why there might be a solution for all prime $n$. However, the interesting part of the pattern is that $n=5cdot 5$ and $n=5cdot 7$ both appear, yet neither $n=3cdot 3$ nor $n=5cdot 5$ appear. Which odd composites admit solutions?
$endgroup$
– Mark Fischler
11 hours ago




$begingroup$
It is easy to picture why there might be a solution for all prime $n$. However, the interesting part of the pattern is that $n=5cdot 5$ and $n=5cdot 7$ both appear, yet neither $n=3cdot 3$ nor $n=5cdot 5$ appear. Which odd composites admit solutions?
$endgroup$
– Mark Fischler
11 hours ago












$begingroup$
There are no even numbers ..
$endgroup$
– Shamim Akhtar
10 hours ago






$begingroup$
There are no even numbers ..
$endgroup$
– Shamim Akhtar
10 hours ago






3




3




$begingroup$
It seems like $n equiv pm1 pmod 6$.
$endgroup$
– md2perpe
10 hours ago




$begingroup$
It seems like $n equiv pm1 pmod 6$.
$endgroup$
– md2perpe
10 hours ago












$begingroup$
Yeah right but why do only specific composites arrive?
$endgroup$
– Shamim Akhtar
10 hours ago




$begingroup$
Yeah right but why do only specific composites arrive?
$endgroup$
– Shamim Akhtar
10 hours ago










3 Answers
3






active

oldest

votes


















11












$begingroup$

$20^6 equiv 13^6 equiv 7^6 equiv 229 bmod 309$, so $20^n equiv 13^n + 7^n mod 309$ if and only if $20^{n+6} equiv 13^{n+6} + 7^{n+6} bmod 309$. Since it does work for $n=1$ and $n=5$, but not for $0$, $2$, $3$ or $4$, we find that $20^n equiv 13^n + 7^n bmod 309$ if and only if $n equiv 1$ or $5 bmod 6$.



All primes except $2$ and $3$ are congruent to $1$ or $5 bmod 6$. $2$ or $3$ can't divide a number congruent to $1$ or $5 bmod 6$, so it's not easy for a small number of this form to be composite. The first few composites are $25 = 5^2$, $35 = 5 cdot 7$, $49 = 7 cdot 7$, $55 = 5 cdot 11$, ...






share|cite|improve this answer











$endgroup$













  • $begingroup$
    +1 really cool.
    $endgroup$
    – Shamim Akhtar
    10 hours ago






  • 2




    $begingroup$
    @ShamimAkhtar If you had entered your sequence $,5,7,11,13,ldots, 65$ into OEIS you'd obtain a unique match: A007310 = Numbers congruent to $1$ or $5$ mod $6$. Once you know that the rest is easy. That's the first method one should try on problems like this (when the pattern is not already clear from prior knowledge).
    $endgroup$
    – Bill Dubuque
    7 hours ago








  • 1




    $begingroup$
    At mathoverflow.net/questions/328782/… there is a proof that in general $(a^2-ab+b^2)$ divides $a^{6k+r} -b^{6k+r} -(a-b)^{6k+r}$ for $r = 1, 5$. The question at hand is a particular case with $a=20, b=13$ where $(a^2-ab+b^2)=309$
    $endgroup$
    – Mark Fischler
    5 hours ago












  • $begingroup$
    @Mark We can also proceed analogously as I do here. (iirc this specific case is done in some other question here).
    $endgroup$
    – Bill Dubuque
    4 hours ago





















1












$begingroup$

So far you have only shown evidence that some of the primes are generated here, not all of them. It's interesting how many there are, though. There are some polynomials that generate an exorbitant number of primes. Here are some examples. You will likely have to do a lot more work than just exploring the idea numerically to prove that all primes are generated. Even if you do, these functions aren't super interesting because they produce non-primes as well. You won't know whether a number that is generated is prime or not until you test it for primality, so it's not like it can be used to generate the $n^{th}$ prime without having to do lots more computation. Quite cool, though.






share|cite|improve this answer









$endgroup$













  • $begingroup$
    Can you explain why this works?
    $endgroup$
    – Shamim Akhtar
    11 hours ago










  • $begingroup$
    I tapped the show more but wolframalpha doesnt show more
    $endgroup$
    – Shamim Akhtar
    11 hours ago






  • 1




    $begingroup$
    I'm afraid I canot
    $endgroup$
    – Michael Stachowsky
    10 hours ago










  • $begingroup$
    Is this a standard question in mse? I too am afraid if any moderator marks this as off topic
    $endgroup$
    – Shamim Akhtar
    10 hours ago






  • 1




    $begingroup$
    It's a perfectly acceptable question, although it's probably much more difficult to answer than we think. As a result, you're probably either not going to get an answer or get a very strange one. I could be wrong, number theory isn't my thing.
    $endgroup$
    – Michael Stachowsky
    10 hours ago



















1












$begingroup$

A trivial observation is that it does not generate all the odd primes: $3$ is missing.



However, I can verify that all primes up to $59359$ (other than $2$ and $3$) are represented.



This has a lot do to with the fact that $(a^2-ab+^2)$ is a apparently factor of $a^n-b^n-(a-b)^n$ for all prime $n>3$. Here, $a=20, b=13$.



But that fact is at least as interesting as your observation, and I don't know how to prove it.






share|cite|improve this answer











$endgroup$













  • $begingroup$
    How can you verify?
    $endgroup$
    – Shamim Akhtar
    10 hours ago






  • 1




    $begingroup$
    Mathematica is very fast in determining residues mod 309.
    $endgroup$
    – Mark Fischler
    10 hours ago












Your Answer








StackExchange.ready(function() {
var channelOptions = {
tags: "".split(" "),
id: "69"
};
initTagRenderer("".split(" "), "".split(" "), channelOptions);

StackExchange.using("externalEditor", function() {
// Have to fire editor after snippets, if snippets enabled
if (StackExchange.settings.snippets.snippetsEnabled) {
StackExchange.using("snippets", function() {
createEditor();
});
}
else {
createEditor();
}
});

function createEditor() {
StackExchange.prepareEditor({
heartbeatType: 'answer',
autoActivateHeartbeat: false,
convertImagesToLinks: true,
noModals: true,
showLowRepImageUploadWarning: true,
reputationToPostImages: 10,
bindNavPrevention: true,
postfix: "",
imageUploader: {
brandingHtml: "Powered by u003ca class="icon-imgur-white" href="https://imgur.com/"u003eu003c/au003e",
contentPolicyHtml: "User contributions licensed under u003ca href="https://creativecommons.org/licenses/by-sa/3.0/"u003ecc by-sa 3.0 with attribution requiredu003c/au003e u003ca href="https://stackoverflow.com/legal/content-policy"u003e(content policy)u003c/au003e",
allowUrls: true
},
noCode: true, onDemand: true,
discardSelector: ".discard-answer"
,immediatelyShowMarkdownHelp:true
});


}
});






Shamim Akhtar is a new contributor. Be nice, and check out our Code of Conduct.










draft saved

draft discarded


















StackExchange.ready(
function () {
StackExchange.openid.initPostLogin('.new-post-login', 'https%3a%2f%2fmath.stackexchange.com%2fquestions%2f3198489%2fwhy-do-all-primes-n3-satisfy-309-mid-20n-13n-7n%23new-answer', 'question_page');
}
);

Post as a guest















Required, but never shown

























3 Answers
3






active

oldest

votes








3 Answers
3






active

oldest

votes









active

oldest

votes






active

oldest

votes









11












$begingroup$

$20^6 equiv 13^6 equiv 7^6 equiv 229 bmod 309$, so $20^n equiv 13^n + 7^n mod 309$ if and only if $20^{n+6} equiv 13^{n+6} + 7^{n+6} bmod 309$. Since it does work for $n=1$ and $n=5$, but not for $0$, $2$, $3$ or $4$, we find that $20^n equiv 13^n + 7^n bmod 309$ if and only if $n equiv 1$ or $5 bmod 6$.



All primes except $2$ and $3$ are congruent to $1$ or $5 bmod 6$. $2$ or $3$ can't divide a number congruent to $1$ or $5 bmod 6$, so it's not easy for a small number of this form to be composite. The first few composites are $25 = 5^2$, $35 = 5 cdot 7$, $49 = 7 cdot 7$, $55 = 5 cdot 11$, ...






share|cite|improve this answer











$endgroup$













  • $begingroup$
    +1 really cool.
    $endgroup$
    – Shamim Akhtar
    10 hours ago






  • 2




    $begingroup$
    @ShamimAkhtar If you had entered your sequence $,5,7,11,13,ldots, 65$ into OEIS you'd obtain a unique match: A007310 = Numbers congruent to $1$ or $5$ mod $6$. Once you know that the rest is easy. That's the first method one should try on problems like this (when the pattern is not already clear from prior knowledge).
    $endgroup$
    – Bill Dubuque
    7 hours ago








  • 1




    $begingroup$
    At mathoverflow.net/questions/328782/… there is a proof that in general $(a^2-ab+b^2)$ divides $a^{6k+r} -b^{6k+r} -(a-b)^{6k+r}$ for $r = 1, 5$. The question at hand is a particular case with $a=20, b=13$ where $(a^2-ab+b^2)=309$
    $endgroup$
    – Mark Fischler
    5 hours ago












  • $begingroup$
    @Mark We can also proceed analogously as I do here. (iirc this specific case is done in some other question here).
    $endgroup$
    – Bill Dubuque
    4 hours ago


















11












$begingroup$

$20^6 equiv 13^6 equiv 7^6 equiv 229 bmod 309$, so $20^n equiv 13^n + 7^n mod 309$ if and only if $20^{n+6} equiv 13^{n+6} + 7^{n+6} bmod 309$. Since it does work for $n=1$ and $n=5$, but not for $0$, $2$, $3$ or $4$, we find that $20^n equiv 13^n + 7^n bmod 309$ if and only if $n equiv 1$ or $5 bmod 6$.



All primes except $2$ and $3$ are congruent to $1$ or $5 bmod 6$. $2$ or $3$ can't divide a number congruent to $1$ or $5 bmod 6$, so it's not easy for a small number of this form to be composite. The first few composites are $25 = 5^2$, $35 = 5 cdot 7$, $49 = 7 cdot 7$, $55 = 5 cdot 11$, ...






share|cite|improve this answer











$endgroup$













  • $begingroup$
    +1 really cool.
    $endgroup$
    – Shamim Akhtar
    10 hours ago






  • 2




    $begingroup$
    @ShamimAkhtar If you had entered your sequence $,5,7,11,13,ldots, 65$ into OEIS you'd obtain a unique match: A007310 = Numbers congruent to $1$ or $5$ mod $6$. Once you know that the rest is easy. That's the first method one should try on problems like this (when the pattern is not already clear from prior knowledge).
    $endgroup$
    – Bill Dubuque
    7 hours ago








  • 1




    $begingroup$
    At mathoverflow.net/questions/328782/… there is a proof that in general $(a^2-ab+b^2)$ divides $a^{6k+r} -b^{6k+r} -(a-b)^{6k+r}$ for $r = 1, 5$. The question at hand is a particular case with $a=20, b=13$ where $(a^2-ab+b^2)=309$
    $endgroup$
    – Mark Fischler
    5 hours ago












  • $begingroup$
    @Mark We can also proceed analogously as I do here. (iirc this specific case is done in some other question here).
    $endgroup$
    – Bill Dubuque
    4 hours ago
















11












11








11





$begingroup$

$20^6 equiv 13^6 equiv 7^6 equiv 229 bmod 309$, so $20^n equiv 13^n + 7^n mod 309$ if and only if $20^{n+6} equiv 13^{n+6} + 7^{n+6} bmod 309$. Since it does work for $n=1$ and $n=5$, but not for $0$, $2$, $3$ or $4$, we find that $20^n equiv 13^n + 7^n bmod 309$ if and only if $n equiv 1$ or $5 bmod 6$.



All primes except $2$ and $3$ are congruent to $1$ or $5 bmod 6$. $2$ or $3$ can't divide a number congruent to $1$ or $5 bmod 6$, so it's not easy for a small number of this form to be composite. The first few composites are $25 = 5^2$, $35 = 5 cdot 7$, $49 = 7 cdot 7$, $55 = 5 cdot 11$, ...






share|cite|improve this answer











$endgroup$



$20^6 equiv 13^6 equiv 7^6 equiv 229 bmod 309$, so $20^n equiv 13^n + 7^n mod 309$ if and only if $20^{n+6} equiv 13^{n+6} + 7^{n+6} bmod 309$. Since it does work for $n=1$ and $n=5$, but not for $0$, $2$, $3$ or $4$, we find that $20^n equiv 13^n + 7^n bmod 309$ if and only if $n equiv 1$ or $5 bmod 6$.



All primes except $2$ and $3$ are congruent to $1$ or $5 bmod 6$. $2$ or $3$ can't divide a number congruent to $1$ or $5 bmod 6$, so it's not easy for a small number of this form to be composite. The first few composites are $25 = 5^2$, $35 = 5 cdot 7$, $49 = 7 cdot 7$, $55 = 5 cdot 11$, ...







share|cite|improve this answer














share|cite|improve this answer



share|cite|improve this answer








edited 10 hours ago

























answered 10 hours ago









Robert IsraelRobert Israel

333k23223484




333k23223484












  • $begingroup$
    +1 really cool.
    $endgroup$
    – Shamim Akhtar
    10 hours ago






  • 2




    $begingroup$
    @ShamimAkhtar If you had entered your sequence $,5,7,11,13,ldots, 65$ into OEIS you'd obtain a unique match: A007310 = Numbers congruent to $1$ or $5$ mod $6$. Once you know that the rest is easy. That's the first method one should try on problems like this (when the pattern is not already clear from prior knowledge).
    $endgroup$
    – Bill Dubuque
    7 hours ago








  • 1




    $begingroup$
    At mathoverflow.net/questions/328782/… there is a proof that in general $(a^2-ab+b^2)$ divides $a^{6k+r} -b^{6k+r} -(a-b)^{6k+r}$ for $r = 1, 5$. The question at hand is a particular case with $a=20, b=13$ where $(a^2-ab+b^2)=309$
    $endgroup$
    – Mark Fischler
    5 hours ago












  • $begingroup$
    @Mark We can also proceed analogously as I do here. (iirc this specific case is done in some other question here).
    $endgroup$
    – Bill Dubuque
    4 hours ago




















  • $begingroup$
    +1 really cool.
    $endgroup$
    – Shamim Akhtar
    10 hours ago






  • 2




    $begingroup$
    @ShamimAkhtar If you had entered your sequence $,5,7,11,13,ldots, 65$ into OEIS you'd obtain a unique match: A007310 = Numbers congruent to $1$ or $5$ mod $6$. Once you know that the rest is easy. That's the first method one should try on problems like this (when the pattern is not already clear from prior knowledge).
    $endgroup$
    – Bill Dubuque
    7 hours ago








  • 1




    $begingroup$
    At mathoverflow.net/questions/328782/… there is a proof that in general $(a^2-ab+b^2)$ divides $a^{6k+r} -b^{6k+r} -(a-b)^{6k+r}$ for $r = 1, 5$. The question at hand is a particular case with $a=20, b=13$ where $(a^2-ab+b^2)=309$
    $endgroup$
    – Mark Fischler
    5 hours ago












  • $begingroup$
    @Mark We can also proceed analogously as I do here. (iirc this specific case is done in some other question here).
    $endgroup$
    – Bill Dubuque
    4 hours ago


















$begingroup$
+1 really cool.
$endgroup$
– Shamim Akhtar
10 hours ago




$begingroup$
+1 really cool.
$endgroup$
– Shamim Akhtar
10 hours ago




2




2




$begingroup$
@ShamimAkhtar If you had entered your sequence $,5,7,11,13,ldots, 65$ into OEIS you'd obtain a unique match: A007310 = Numbers congruent to $1$ or $5$ mod $6$. Once you know that the rest is easy. That's the first method one should try on problems like this (when the pattern is not already clear from prior knowledge).
$endgroup$
– Bill Dubuque
7 hours ago






$begingroup$
@ShamimAkhtar If you had entered your sequence $,5,7,11,13,ldots, 65$ into OEIS you'd obtain a unique match: A007310 = Numbers congruent to $1$ or $5$ mod $6$. Once you know that the rest is easy. That's the first method one should try on problems like this (when the pattern is not already clear from prior knowledge).
$endgroup$
– Bill Dubuque
7 hours ago






1




1




$begingroup$
At mathoverflow.net/questions/328782/… there is a proof that in general $(a^2-ab+b^2)$ divides $a^{6k+r} -b^{6k+r} -(a-b)^{6k+r}$ for $r = 1, 5$. The question at hand is a particular case with $a=20, b=13$ where $(a^2-ab+b^2)=309$
$endgroup$
– Mark Fischler
5 hours ago






$begingroup$
At mathoverflow.net/questions/328782/… there is a proof that in general $(a^2-ab+b^2)$ divides $a^{6k+r} -b^{6k+r} -(a-b)^{6k+r}$ for $r = 1, 5$. The question at hand is a particular case with $a=20, b=13$ where $(a^2-ab+b^2)=309$
$endgroup$
– Mark Fischler
5 hours ago














$begingroup$
@Mark We can also proceed analogously as I do here. (iirc this specific case is done in some other question here).
$endgroup$
– Bill Dubuque
4 hours ago






$begingroup$
@Mark We can also proceed analogously as I do here. (iirc this specific case is done in some other question here).
$endgroup$
– Bill Dubuque
4 hours ago













1












$begingroup$

So far you have only shown evidence that some of the primes are generated here, not all of them. It's interesting how many there are, though. There are some polynomials that generate an exorbitant number of primes. Here are some examples. You will likely have to do a lot more work than just exploring the idea numerically to prove that all primes are generated. Even if you do, these functions aren't super interesting because they produce non-primes as well. You won't know whether a number that is generated is prime or not until you test it for primality, so it's not like it can be used to generate the $n^{th}$ prime without having to do lots more computation. Quite cool, though.






share|cite|improve this answer









$endgroup$













  • $begingroup$
    Can you explain why this works?
    $endgroup$
    – Shamim Akhtar
    11 hours ago










  • $begingroup$
    I tapped the show more but wolframalpha doesnt show more
    $endgroup$
    – Shamim Akhtar
    11 hours ago






  • 1




    $begingroup$
    I'm afraid I canot
    $endgroup$
    – Michael Stachowsky
    10 hours ago










  • $begingroup$
    Is this a standard question in mse? I too am afraid if any moderator marks this as off topic
    $endgroup$
    – Shamim Akhtar
    10 hours ago






  • 1




    $begingroup$
    It's a perfectly acceptable question, although it's probably much more difficult to answer than we think. As a result, you're probably either not going to get an answer or get a very strange one. I could be wrong, number theory isn't my thing.
    $endgroup$
    – Michael Stachowsky
    10 hours ago
















1












$begingroup$

So far you have only shown evidence that some of the primes are generated here, not all of them. It's interesting how many there are, though. There are some polynomials that generate an exorbitant number of primes. Here are some examples. You will likely have to do a lot more work than just exploring the idea numerically to prove that all primes are generated. Even if you do, these functions aren't super interesting because they produce non-primes as well. You won't know whether a number that is generated is prime or not until you test it for primality, so it's not like it can be used to generate the $n^{th}$ prime without having to do lots more computation. Quite cool, though.






share|cite|improve this answer









$endgroup$













  • $begingroup$
    Can you explain why this works?
    $endgroup$
    – Shamim Akhtar
    11 hours ago










  • $begingroup$
    I tapped the show more but wolframalpha doesnt show more
    $endgroup$
    – Shamim Akhtar
    11 hours ago






  • 1




    $begingroup$
    I'm afraid I canot
    $endgroup$
    – Michael Stachowsky
    10 hours ago










  • $begingroup$
    Is this a standard question in mse? I too am afraid if any moderator marks this as off topic
    $endgroup$
    – Shamim Akhtar
    10 hours ago






  • 1




    $begingroup$
    It's a perfectly acceptable question, although it's probably much more difficult to answer than we think. As a result, you're probably either not going to get an answer or get a very strange one. I could be wrong, number theory isn't my thing.
    $endgroup$
    – Michael Stachowsky
    10 hours ago














1












1








1





$begingroup$

So far you have only shown evidence that some of the primes are generated here, not all of them. It's interesting how many there are, though. There are some polynomials that generate an exorbitant number of primes. Here are some examples. You will likely have to do a lot more work than just exploring the idea numerically to prove that all primes are generated. Even if you do, these functions aren't super interesting because they produce non-primes as well. You won't know whether a number that is generated is prime or not until you test it for primality, so it's not like it can be used to generate the $n^{th}$ prime without having to do lots more computation. Quite cool, though.






share|cite|improve this answer









$endgroup$



So far you have only shown evidence that some of the primes are generated here, not all of them. It's interesting how many there are, though. There are some polynomials that generate an exorbitant number of primes. Here are some examples. You will likely have to do a lot more work than just exploring the idea numerically to prove that all primes are generated. Even if you do, these functions aren't super interesting because they produce non-primes as well. You won't know whether a number that is generated is prime or not until you test it for primality, so it's not like it can be used to generate the $n^{th}$ prime without having to do lots more computation. Quite cool, though.







share|cite|improve this answer












share|cite|improve this answer



share|cite|improve this answer










answered 11 hours ago









Michael StachowskyMichael Stachowsky

1,409418




1,409418












  • $begingroup$
    Can you explain why this works?
    $endgroup$
    – Shamim Akhtar
    11 hours ago










  • $begingroup$
    I tapped the show more but wolframalpha doesnt show more
    $endgroup$
    – Shamim Akhtar
    11 hours ago






  • 1




    $begingroup$
    I'm afraid I canot
    $endgroup$
    – Michael Stachowsky
    10 hours ago










  • $begingroup$
    Is this a standard question in mse? I too am afraid if any moderator marks this as off topic
    $endgroup$
    – Shamim Akhtar
    10 hours ago






  • 1




    $begingroup$
    It's a perfectly acceptable question, although it's probably much more difficult to answer than we think. As a result, you're probably either not going to get an answer or get a very strange one. I could be wrong, number theory isn't my thing.
    $endgroup$
    – Michael Stachowsky
    10 hours ago


















  • $begingroup$
    Can you explain why this works?
    $endgroup$
    – Shamim Akhtar
    11 hours ago










  • $begingroup$
    I tapped the show more but wolframalpha doesnt show more
    $endgroup$
    – Shamim Akhtar
    11 hours ago






  • 1




    $begingroup$
    I'm afraid I canot
    $endgroup$
    – Michael Stachowsky
    10 hours ago










  • $begingroup$
    Is this a standard question in mse? I too am afraid if any moderator marks this as off topic
    $endgroup$
    – Shamim Akhtar
    10 hours ago






  • 1




    $begingroup$
    It's a perfectly acceptable question, although it's probably much more difficult to answer than we think. As a result, you're probably either not going to get an answer or get a very strange one. I could be wrong, number theory isn't my thing.
    $endgroup$
    – Michael Stachowsky
    10 hours ago
















$begingroup$
Can you explain why this works?
$endgroup$
– Shamim Akhtar
11 hours ago




$begingroup$
Can you explain why this works?
$endgroup$
– Shamim Akhtar
11 hours ago












$begingroup$
I tapped the show more but wolframalpha doesnt show more
$endgroup$
– Shamim Akhtar
11 hours ago




$begingroup$
I tapped the show more but wolframalpha doesnt show more
$endgroup$
– Shamim Akhtar
11 hours ago




1




1




$begingroup$
I'm afraid I canot
$endgroup$
– Michael Stachowsky
10 hours ago




$begingroup$
I'm afraid I canot
$endgroup$
– Michael Stachowsky
10 hours ago












$begingroup$
Is this a standard question in mse? I too am afraid if any moderator marks this as off topic
$endgroup$
– Shamim Akhtar
10 hours ago




$begingroup$
Is this a standard question in mse? I too am afraid if any moderator marks this as off topic
$endgroup$
– Shamim Akhtar
10 hours ago




1




1




$begingroup$
It's a perfectly acceptable question, although it's probably much more difficult to answer than we think. As a result, you're probably either not going to get an answer or get a very strange one. I could be wrong, number theory isn't my thing.
$endgroup$
– Michael Stachowsky
10 hours ago




$begingroup$
It's a perfectly acceptable question, although it's probably much more difficult to answer than we think. As a result, you're probably either not going to get an answer or get a very strange one. I could be wrong, number theory isn't my thing.
$endgroup$
– Michael Stachowsky
10 hours ago











1












$begingroup$

A trivial observation is that it does not generate all the odd primes: $3$ is missing.



However, I can verify that all primes up to $59359$ (other than $2$ and $3$) are represented.



This has a lot do to with the fact that $(a^2-ab+^2)$ is a apparently factor of $a^n-b^n-(a-b)^n$ for all prime $n>3$. Here, $a=20, b=13$.



But that fact is at least as interesting as your observation, and I don't know how to prove it.






share|cite|improve this answer











$endgroup$













  • $begingroup$
    How can you verify?
    $endgroup$
    – Shamim Akhtar
    10 hours ago






  • 1




    $begingroup$
    Mathematica is very fast in determining residues mod 309.
    $endgroup$
    – Mark Fischler
    10 hours ago
















1












$begingroup$

A trivial observation is that it does not generate all the odd primes: $3$ is missing.



However, I can verify that all primes up to $59359$ (other than $2$ and $3$) are represented.



This has a lot do to with the fact that $(a^2-ab+^2)$ is a apparently factor of $a^n-b^n-(a-b)^n$ for all prime $n>3$. Here, $a=20, b=13$.



But that fact is at least as interesting as your observation, and I don't know how to prove it.






share|cite|improve this answer











$endgroup$













  • $begingroup$
    How can you verify?
    $endgroup$
    – Shamim Akhtar
    10 hours ago






  • 1




    $begingroup$
    Mathematica is very fast in determining residues mod 309.
    $endgroup$
    – Mark Fischler
    10 hours ago














1












1








1





$begingroup$

A trivial observation is that it does not generate all the odd primes: $3$ is missing.



However, I can verify that all primes up to $59359$ (other than $2$ and $3$) are represented.



This has a lot do to with the fact that $(a^2-ab+^2)$ is a apparently factor of $a^n-b^n-(a-b)^n$ for all prime $n>3$. Here, $a=20, b=13$.



But that fact is at least as interesting as your observation, and I don't know how to prove it.






share|cite|improve this answer











$endgroup$



A trivial observation is that it does not generate all the odd primes: $3$ is missing.



However, I can verify that all primes up to $59359$ (other than $2$ and $3$) are represented.



This has a lot do to with the fact that $(a^2-ab+^2)$ is a apparently factor of $a^n-b^n-(a-b)^n$ for all prime $n>3$. Here, $a=20, b=13$.



But that fact is at least as interesting as your observation, and I don't know how to prove it.







share|cite|improve this answer














share|cite|improve this answer



share|cite|improve this answer








edited 10 hours ago

























answered 10 hours ago









Mark FischlerMark Fischler

34.6k12552




34.6k12552












  • $begingroup$
    How can you verify?
    $endgroup$
    – Shamim Akhtar
    10 hours ago






  • 1




    $begingroup$
    Mathematica is very fast in determining residues mod 309.
    $endgroup$
    – Mark Fischler
    10 hours ago


















  • $begingroup$
    How can you verify?
    $endgroup$
    – Shamim Akhtar
    10 hours ago






  • 1




    $begingroup$
    Mathematica is very fast in determining residues mod 309.
    $endgroup$
    – Mark Fischler
    10 hours ago
















$begingroup$
How can you verify?
$endgroup$
– Shamim Akhtar
10 hours ago




$begingroup$
How can you verify?
$endgroup$
– Shamim Akhtar
10 hours ago




1




1




$begingroup$
Mathematica is very fast in determining residues mod 309.
$endgroup$
– Mark Fischler
10 hours ago




$begingroup$
Mathematica is very fast in determining residues mod 309.
$endgroup$
– Mark Fischler
10 hours ago










Shamim Akhtar is a new contributor. Be nice, and check out our Code of Conduct.










draft saved

draft discarded


















Shamim Akhtar is a new contributor. Be nice, and check out our Code of Conduct.













Shamim Akhtar is a new contributor. Be nice, and check out our Code of Conduct.












Shamim Akhtar is a new contributor. Be nice, and check out our Code of Conduct.
















Thanks for contributing an answer to Mathematics Stack Exchange!


  • Please be sure to answer the question. Provide details and share your research!

But avoid



  • Asking for help, clarification, or responding to other answers.

  • Making statements based on opinion; back them up with references or personal experience.


Use MathJax to format equations. MathJax reference.


To learn more, see our tips on writing great answers.




draft saved


draft discarded














StackExchange.ready(
function () {
StackExchange.openid.initPostLogin('.new-post-login', 'https%3a%2f%2fmath.stackexchange.com%2fquestions%2f3198489%2fwhy-do-all-primes-n3-satisfy-309-mid-20n-13n-7n%23new-answer', 'question_page');
}
);

Post as a guest















Required, but never shown





















































Required, but never shown














Required, but never shown












Required, but never shown







Required, but never shown

































Required, but never shown














Required, but never shown












Required, but never shown







Required, but never shown







Popular posts from this blog

Why do type traits not work with types in namespace scope?What are POD types in C++?Why can templates only be...

Will tsunami waves travel forever if there was no land?Why do tsunami waves begin with the water flowing away...

Should I use Docker or LXD?How to cache (more) data on SSD/RAM to avoid spin up?Unable to get Windows File...